Business Finance Chapter 11

अब Quizwiz के साथ अपने होमवर्क और परीक्षाओं को एस करें!

A stock is expected to return 13 percent in an economic boom, 10 percent in a normal economy, and 3 percent in a recessionary economy. Which one of the following will lower the overall expected rate of return on this stock?

A decrease in the probability of an economic boom

Which statement is correct?

A security with a beta of 1.54 will plot on the security market line if it is correctly priced.

Which one of the following is the best example of unsystematic risk?

A warehouse fire

Which statement is correct?

An underpriced security will plot above the security market line.

Which one of the following is the minimum required rate of return on a new investment that makes that investment attractive?

Cost of capital

Which one of these is the best example of systematic risk?

Decrease in gross domestic product

Mary owns a risky stock and anticipates earning 16.5 percent on her investment in that stock. Which one of the following best describes the 16.5 percent rate?

Expected return

A portfolio is:

Group of assets held by an investor

Which one of the following statements is correct?

If a risky security is correctly priced, its expected risk premium will be positive.

World United stock currently plots on the security market line and has a beta of 1.04. Which one of the following will increase that stock's rate of return without affecting the risk level of the stock, all else constant?

Increase in the market risk-to-reward ratio

Stock A comprises 28 percent of Susan's portfolio. Which one of the following terms applies to the 28 percent?

Portfolio weight

Which one of the following is the best example of an announcement that is most apt to result in an unexpected return?

Statement by a firm that it has just discovered a manufacturing defect and is recalling its product

Which one of the following best exemplifies unsystematic risk?

Unexpected increase in the variable costs for a firm

Which one of the following portfolios will have a beta of zero?

a portfolio comprised solely of U.S. Treasury bills

Systematic risk is:

also known as "market risk" or "un-diversifiable risk", is the uncertainty inherent to the entire market or entire market segment. OR unexpected increase in the variable costs for a firm

Systematic risk is defined as:

any risk that affects a large number of assets

The amount of systematic risk present in a particular risky asset relative to that in an average risky asset is measured by the:

beta coefficient.

The capital asset pricing model:

considers the relationship between the fluctuations in a security's returns versus the market's returns.

The security market line is defined as a positively sloped straight line that displays the relationship between the:

expected return and beta of either a security or a portfolio.

The security market line is a linear function that is graphed by plotting data points based on the relationship between the:

expected return and beta.

The systematic risk principle states that the expected return on a risky asset depends only on the asset's ________ risk.

market

Unsystematic risk can be defined by all of the following except:

market risk

The slope of the security market line represents the:

market risk premium.

The addition of a risky security to a fully diversified portfolio:

may or may not affect the portfolio beta.

If a security plots to the right and below the security market line, then the security has ____ systematic risk than the market and is ____.

more; overpriced

Assume you own a portfolio of diverse securities which are each correctly priced. Given this, the reward-to-risk ratio:

of each security must equal the slope of the security market line.

According to the capital asset pricing model, the expected return on a security will be affected by all of the following except the:

security's standard deviation.

The expected return on a security is not affected by the:

security's unique risks.

Portfolio diversification eliminates:

unsystematic risk.

A portfolio is comprised of 35 securities with varying betas. The lowest beta for an individual security is .74 and the highest of the security betas of 1.51. Given this information, you know that the portfolio beta:

will be greater than or equal to .74 but less than or equal to 1.51.

Diversifying a portfolio across various sectors and industries might do more than one of the following. However, this diversification must do which one of the following?

Reduce the portfolio's unique risks

The risk premium for an individual security is based on which one of the following types of risk?

Systematic

Which one of the following represents the amount of compensation an investor should expect to receive for accepting the unsystematic risk associated with an individual security?

Zero

Which one of the following is the computation of the risk premium for an individual security? E(R) is the expected return on the security, Rf is the risk-free rate, β is the security's beta, and E(RM) is the expected rate of return on the market.

β[E(RM) -Rf]

Standard deviation measures _____ risk while beta measures _____ risk.

total; systematic

Which term best refers to the practice of investing in a variety of diverse assets as a means of reducing risk?

Diversification

Which one of these represents systematic risk?

Increase in consumption created by a reduction in personal tax rates

Which one of the following is the vertical intercept of the security market line?

Risk-free rate

Consider a portfolio comprised of four risky securities. Assume the economy has three economic states with varying probabilities of occurrence. Which one of the following will guarantee that the portfolio variance will equal zero?

The portfolio expected rate of return must be the same for each economic state.

The expected rate of return on Delaware Shores stock is based on three possible states of the economy. These states are boom, normal, and recession which have probabilities of occurrence of 20 percent, 75 percent, and 5 percent, respectively. Which one of the following statements is correct concerning the variance of the returns on this stock?

The variance must be positive provided that each state of the economy produces a different expected rate of return

Which statement is true?

The weights of the securities held in any portfolio must equal 1.0.

For a risky security to have a positive expected return but less risk than the overall market, the security must have a beta:

that is > 0 but < 1.

The beta of a risky portfolio cannot be less than _____ nor greater than ____.

the lowest individual beta in the portfolio; the highest individual beta in the portfolio


संबंधित स्टडी सेट्स

CH 23 - Decentralized Organizations

View Set

Latin America & the Caribbean Test Review

View Set

(3) Chapter 28: The Bereaved Individual **

View Set

Using the Wave Formula (Speed = Wavelength x Frequency)

View Set

6b. Los días de la semana (Days of the Week)

View Set